Ich üb mal wieder für die Olympiade - Seite 5

Neue Frage »

AD Auf diesen Beitrag antworten »

Zitat:
Original von Sciencefreak
Ich bin der Meinung, dass es wahrscheinlich noch eine sinnvollere Variante gibt, denn spätestens bei n>10 kann man das nicht mehr schriftlich auf die Reihe bekommen.

Wieso? Ich empfinde das als eine sehr kurz aufschreibbare Variante - es sei denn, du hast eine andere Idee als ich.
Sciencefreak Auf diesen Beitrag antworten »

ich gehe erst mal über die Veränderung von n. Wenn z durch n teilbar sein soll, so ist z auch durch m teilbar, wenn m die Zahl ist, die in der Primfaktorzerlegung genauso viele Zweien wie Fünfen hat. Somit kann ich erst mal sagen, wie viele Nullen am Ende stehen müssen. Und dann nehme ich die letzte Ziffer von m die nicht 0 ist und das ist dann die Ziffer die vor den ganzen Nullen steht. Und dann beginnt meine eigentliche komische Variante. Dann verändere ich die Zahl m so, dass sie nicht mehr durch 10 teilbar ist (einfach durch mehrfaches teilen durch 10). Mit dieser neuen Zahl a verfahre ich dann wie folgt. Zuerst nehme ich a, dann addiere ich so lange 10*a dazu, bis die Zehnerstelle dieser Summe die gewünschte Ziffer ist. Dann addiere ich 100*a bis die Hunderterstelle die gewünschte Ziffer ist ...
Dabei verändert sich natürich immer der "Rest" vor den richtigen Ziffern, aber er bleibt immer kleiner als a und falls man mit dieser Methode 2-mal auf den gleichen "Rest" kommt oder wenn der "Rest" aus den gewünschten Ziffern besteht, dann bin ich fertig. bei dem gleichen "Rest muss ich natürlich nur noch die Zahl wo der Rest zuerst aufgetreten ist mal die richtige Zehnerpotenz abziehen und habe dann auch eine Zahl, die durch a teilbar ist. Da es nur n natürliche Zahlen kleiner n gibt ist diese Methode irgendwann zu Ende. Dann kann man am Ende noch die gewünschten Nullen dranhängen und man ist fertig. Natürlich würde ich das bei einer Matheolympiade nicht so schreiben, aber meine Idee sollte man verstehen
AD Auf diesen Beitrag antworten »

Ui, ganz schön lang - ich lese es mir noch mal durch. Da ist meine Variante doch ganz anders:

Sei die Zahl, die aus genau k Einsen besteht. Nun betrachte ich die Zahlen modulo n, dann sind nach Schubfachprinzip mindestens zwei dieser Reste gleich, das seien die von und mit .

Dann erfüllt die Aufgabenstellung.
Sciencefreak Auf diesen Beitrag antworten »

Ich wusste doch das meine Variante zu umständlich ist. Aber ich habe da ja immer Zeit genug und richtig ist meine an sich auch
Steve_FL Auf diesen Beitrag antworten »

Dazu gibt es eine ähnliche Aufgabe Augenzwinkern

Zeige, dass es für jedes n, das weder durch 2 noch durch 5 teilbar, ein vielfaches von n gibt, das nur aus 1en besteht. (Oder so ähnlicher Wortlaut...)

Ist eigentlich nicht soooo schwer Augenzwinkern

Übrigens, kleine Nebeninfo:
Ich hab die Qualifikation bestanden Augenzwinkern
AD Auf diesen Beitrag antworten »

@Steve

Gratulation! Freude

Und natürlich Prost Prost , solange es nicht einen Tag vor der Klausur ist. Augenzwinkern


@Sciencefreak

Ich hab's inzwischen durchgelesen. Deine Variante ist doch gar nicht so anders als meine: Auch du fährst ja fort, bis der gleiche "Rest" mehrfach auftaucht - bei dir sind der "Rest" halt nur die Ziffern vor der Periode.
 
 
Sciencefreak Auf diesen Beitrag antworten »

Aber deine Variante sieht erst mal besser aus und ist auch leichter zu erklären.
@Steve:Gratulation auch von mir. Ich hoffe ich komme dieses Jahr etwas weiter. Und war das schon die Qualifikation zur Imo oder nur eine der vielen davor? Ich glaub ich muss 4 oder 5 Runden überstehen, wenn ich zur IMO will und bin letztes Jahr an der ersten gescheitert
Steve_FL Auf diesen Beitrag antworten »

ich musste nur 2 überstehen Augenzwinkern In der Schweiz muss man normalerweise 3 oder 4 glaub ich smile

Ja, das war die zur IMO. In Liechtenstein (das erste Mal, dass ein Team aus unserem Land geht) wars etwas einfacher. Hier ist ein Link zur Prüfung, die ich schreiben musste:
Klick

mfg
Sciencefreak Auf diesen Beitrag antworten »

Also die erste kommt mir sehr bekannt vor, ich glaube die habe ich schon mal gerechnet. Geometrie kann ich nie leiden. 3. und 4. hätte ich erst mal ein paar Ideen zu und zu 5. habe ich wahrscheinlich auch keinen Plan
AD Auf diesen Beitrag antworten »

@Steve

Mal eine Frage zu Aufgabe 4: Größer oder gleich 14 ist klar, und man kann eine Konfiguration für 15 angeben. Du kennst ja wahrscheinlich die offizielle Lösung - gibt es eine elegante Variante, die Nichtrealisierbarkeit von 14 nachzuweisen?

Und was die Aufgaben 1 und 3 betrifft: Da hast du dich mit Ungleichungen und Zahlentheorie ja gerade richtig vorbereitet. Augenzwinkern


EDIT: Ähmm, hat sich erledigt, die Frage zu Aufgabe 4.
Steve_FL Auf diesen Beitrag antworten »

http://www.imosuisse.ch/index.php?TPL=121&x121_ITEMID=198

das sind die Lösungen Augenzwinkern

Aufgabe 1 und 2 hab ich komplett richtig. Bei der 3. hab ich eine Lösung zuviel angegeben (vergessen, diese auszuschliessen durch die Nebenbediengung mit d8 und d9...) da gabs nur 6 von 7 Punkten und bei der 4. hatte ich 3 Punkte.
Insgesamt also 23 von 35.
AD Auf diesen Beitrag antworten »

Wirklich nett, die Leute da bei euch - reiten nicht auf Formalitäten herum:

Zitat:
Auszug aus Musterlösung zu Aufgabe 5
Dass dies stimmt, kann man sich mit vollständiger Induktion klar machen. Ein expliziter Induktionsbeweis wird nicht verlangt, weil man, um auf diese Formeln zu kommen, genau die Ideen haben muss, die auch für den Induktionsbeweis benötigt werden.

smile
Mathespezialschüler Auf diesen Beitrag antworten »

Zur Aufgabe 2:
Die Punkte D und E müssen doch gar nicht auf der Strecke BC liegen oder sollen sie nur auf der verlängerten Geraden liegen? verwirrt

PS: @Steve
"Nachträglich" nochmal Glückwunsch von mir! smile
Und viel Spaß bei der IMO! Rock
Sciencefreak Auf diesen Beitrag antworten »

Mal eine andere Frage. Viele Matheolympiade aufgaben laufen ja darauf hinaus, dass das Produkt von n positiven Zahlen mit fester Summe genau dann am größten ist, wenn alle n Zahlen gleich sind. Gibt es dafür irgendeinen kurzen Beweis?
Oder die Quadrate von n Zahlen mit fester Summe sind dann am kleinsten wenn alle Zahlen gleich sind
AD Auf diesen Beitrag antworten »

Erste Frage AMGM, zweite Frage AMQM .

AM = arithmetisches Mittel
GM = geometrisches Mittel
QM = quadratisches Mittel

Gemeint sind natürlich die jeweiligen Ungleichungen, bei denen genau dann Gleichheit auftritt, wenn die n dort einbezogenen Zahlen einander gleich sind.
Sciencefreak Auf diesen Beitrag antworten »

Darauf hätte ich auch selber kommen können Hammer
Sciencefreak Auf diesen Beitrag antworten »

Mir will mal wieder nichts für diese Aufgabe einfallen
Zitat:
Ermittle alle reelen x für die gilt


Ich bin jetzt zu der Meinung gekommen das 0 die einzige Lösung ist, aber bei der Begründung happert es etwas. Für ganze Zahlen wäre das ja kein Problem, da man den Bereich leicht einschränken kann, das Problem ist nur zwischen 2 unterschiedlichen rationalen Zahlen liegen unendlich viele reele Zahlen.
AD Auf diesen Beitrag antworten »

Schreib die Gleichung mal "schön symmetrisch" mit . smile
Sciencefreak Auf diesen Beitrag antworten »


Ja soweit war ich schon mal gewesen. Da konnte man auch schön erkennen, dass a=b=c gilt, aber gerade an dem Beweis das dies gilt bin ich irgendwie gescheitert. Diese Substitution mus ja sofort ins Auge springen, schon wegen den 3 Quadratzahlen als Basis.
edit:Das dumme kommt erst mit der Fallunterscheidung. Also es gilt entweder
oder oder

Bei dem 2.Fall kommt man schnell zu einer Lösung aber die anderen beiden Fälle bereiten mir Kopfzerbrechen und was meintest du mit symetrisch
AD Auf diesen Beitrag antworten »

Das war schon das, was ich meinte. Und dann mit 2 multiplizieren:

Sciencefreak Auf diesen Beitrag antworten »

Hammer Hammer Hammer
Und wer soll auf so etwas kommen? Ich hatte schon überlegt was man mit den ab,ac und bc anfangen kann, aber wenn man dann erst noch mit 2 multiplizieren muss. Bei deiner Schreibung ist es jetzt natürlich klar
Sciencefreak Auf diesen Beitrag antworten »

Ich hab jetzt einfach mal die Aufgaben der vorigen Jahre durchgerechnet , aber ich habe bei der dritten Aufgabe hier keine richtige Idee
http://www.mathematik-olympiaden.de/Aufgaben/40/4/40104a.pdf
Tut mir Leid, dass ich sienicht gleich hier stellen kann, aber die Sachen mit dem Formeleditor lassen sich schlecht kopieren.
Also meinerste Gedanke wäre das da eine Strecke entstehen müsste, weil mich das ganze irgendwie an Inversion am Kreis erinnert hat. Aber wie begründe ich das? Oder ist mein Gedanke schon falsch?

Und wie steht es mit den anderen Aufgaben die ich gerechnet habe? Ich habe zwar wieder halbweg vernünftige Ergebnisse aber vielleicht gibt es wieder einen etwas kürzeren Lösungsweg.
401041
a)es gibt gerade und ungerade Zahlen







Entweder n oder n+1 ist duch 2 teilbar somit ist das Produkt durch 2 teilbar
b)
Ich betrachte die Varianten für alle Ziffern extra. Erst mal gibt es keine 2-stellige Lösung(Quadratzaheln aufschreiben)
dann sind die letzten 3 Ziffern entweder

111 (110 ist nicht durch 8 teilbar)
222 (222 ist nicht durch 4 teilbar)
333 (332 ist nicht durch 8 teilbar)
444 (444 ist zwar durch 4 teilbar, aber wenn das eine Lösung wäre,so wäre die Quadratzahl auch als darstellbar also wiederspruch zur ersten Variante)
555 (554 ist nicht durch 8 teilbar)
666 (666 ist nicht durch 4 teilbar)
777 (776 ist durch 8 teilbar, aber keine Quadratzahl endet auf 7(letzte Stellen durchprobieren)
888 (ist durch 4 teilbar, aber analog zu 444 auf 222 reduzierbar also widerspruch)
999 (998 nicht durch 8 teilbar)
000 muss nicht betrachtet werden



401042


Der letzte Term ist für n=1030 zum Beispeil größer als 10
Somit ist Aussage A falsch und die beiden anderen richtig

Edit:Latex mag den Code über 2 Zeilen nicht, deshalb 2 extra Zeilen bei Aufgabe 401042
Edit2:Geht vielleicht dch, Latex wollte nur eine geschweifte Klammer mehr haben
Edit3:Meine Lösungen sind zwar nicht gerade ausformuliert, aber man sollte sie verstehen können, wenn jemand etwas nicht versteht dann einfach hinschreiben und ich versuch es etwas genauer zu erklären
Egal Auf diesen Beitrag antworten »

Also bei der 401042 würd mich schon interessieren wieso die erste abschätzung gilt?
AD Auf diesen Beitrag antworten »

Die Schwierigkeit bei Aufgabe 401042 besteht für mich eigentlich darin, das möglichst elementar zu formulieren, da man von Zehntklässlern kaum Grenzwertrechnung oder Integralrechnung verlangen kann. Aber Sciencefreak wird bei seinem Wissen ja mit ähnlichen Problemen zu kämpfen haben, weswegen ich mich eigentlich wundere, dass er nicht schon lange in der 11-13 startet.

@Egal

Zur Abschätzung:

Sciencefreak Auf diesen Beitrag antworten »

Ja die Sache war mir etwas kompliziert um sie sinnvoll aufzuschreiben, aber das werde ich bei der Matheolympoiade dann selbstverständlich tun. Meine Überlegungen decken sich auch mit der von Arthur, nur dass ich das wahrscheinlich nicht so schön hätte schreiben können.
Zitat:
Aber Sciencefreak wird bei seinem Wissen ja mit ähnlichen Problemen zu kämpfen haben, weswegen ich mich eigentlich wundere, dass er nicht schon lange in der 11-13 startet.

ich habe probleme und soll in einer höheren Klasse starten?
Wieso sollte ich, wenn ich Probleme habe sogar in noch einer höheren Klassenstufe starten? Und da habe ich dann vielleicht Probleme. Ich kann zwar das ganze im Allgemeinen, aber irgendwo fehlt mir dann halt etwas Übung und es ist ja auch nicht so wie bei der Stadtolympiade wo es jetzt eigentlich nur noch darum geht wie viele Punkte ich vor dem 2.Platz bin(dieses Jahr waren es schon 8, letztes Jahr waren es bloß 6 Punkte, was bei einer Maximalpunktzahl von 40 Punkten eine ganze Menge ist). Ich hatte auch letztes Jahr bei der Bundesrunde nur einen 3.Preis bekommen, weil ich ein paar dumme Fehler gemacht hatte

Und wie steht es mit der 3.Aufgabe? Da habe ich immer noch ein Problem.
AD Auf diesen Beitrag antworten »

Ich glaub, da hast du was gründlich missverstanden: Meine Anspielung bezog sich darauf, dass du viel mehr Mathematikwissen besitzt, als deiner Klassenstufe entspricht - z.B. eben Grenzwerte und Differenzial- und Integralrechnung (mit diesem Wissen lässt sich z.B. die Summe durch ein Integral über 1/x abschätzen). Außerdem erhöht ein "Frühstart" die Chancen, eher bzw. auch irgendwann bei der IMO zu starten - war zumindest früher so. Vielleicht ist das heute ja anders.
Sciencefreak Auf diesen Beitrag antworten »

Bei uns hat man halt nur die normalen Chancen. Ein Frühstart hat nur den Effekt, dass man sich früher für einen davon vollkommen unabhängigen IMO-Auswahlvorgang beteiligen kann und den habe ich ja letztes Jahr gehörig vergeigt und aber immerhin noch besser als mein Freund, der bei der Bundesrunde im Gegensatz zu mir einen 1.Preis bekommen hat. Vorteile hat der Frühstart bloß, wenn du früher weiterkommen willst. Ich habe zum Beispiel in der 2.Klasse bei der 3.Klasse teilgenommen, weil es für die 2.Klasse keine Stadtolympiade gibt. Dann habe ich in der 5.Klasse bei der 6.Klasse teilgenommen um zur Landesolympiade zu kommen, was ich auch geschafft habe und dort habe ich gelert, was man alles falsch machen kann. Und bei der Bundesrunde bin ich schon über einen ersten Preis in meiner Klassenstufe sehr glücklich aber ich bin da nicht ganz so optimistisch.
Und mit der Differenzialrechnung und Integralrechnung habe ich erst vor einigen Wochen angefangen, da der Matheunterricht total langweilig ist
Und Grenzwertbestimmung und so etwas mache ich auch eher nach Bauchgefühl und habe so etwas nie richtig gelernt, aber ich weiß sehr gut, wie ich das begründe. Außerdem gibt es in meiner Klassenstufe genug Konkurenz
AD Auf diesen Beitrag antworten »

Zu 401043:

Zeichne mal den durch Z verlaufenden Durchmesser des Ausgangskreises ein - ein Durchmesser der Länge übrigens.
Mathespezialschüler Auf diesen Beitrag antworten »

Zitat:
Original von Sciencefreak

Und woher hast du diese Abschätzung?
Zumal der letzte Term für n=1030 nur knapp größer als 5 wird, aber nicht größer als 10! Da man in Arthurs Abschätzung für n>1 getrost aus dem ein machen kann, muss nur gelten und mit dieser Abschätzung kommt man auf . Du musst ja nur eine Zahl zwischen 1000² und 1050² und nicht zwischen 1000 und 1050 finden! Solch eine ist dann z.B. . Augenzwinkern

@Arthur
Meinst du vielleicht folgendes?
Für die Integralabschätzung fällt mir spontan



ein, womit man mit TR z.B. auf kommt, ohne TR immerhin auch auf oder auch , mit der die Aussage C gesichert ist.
AD Auf diesen Beitrag antworten »

@MSS

Ja, genau sowas hatte ich im Sinn. Aber wie gesagt, notwendig ist Integralrechnung zur Lösung hier nicht, es geht auch ganz gut ohne.
Sciencefreak Auf diesen Beitrag antworten »

Ja ich hatte schon einen Schritt weiter gedacht gehabt, denn ich habe ln(n^2)=2ln(n) genommen und die 2 mit deeem 1/2 gekürzt. Hab da was falsch abgeschrieben gehabt.
Zu der 3.Aufgabe komme ich auf den Kathetensatz und demzufolge liegen die Punkte P' auf der Tangente an Q und Q ist dabei der 2.Schnittpunkt des Durchmessers durch Z. Natürlich nur auf dem auf einem teil dieser Gerade, welches durch die Geraden durch Z und A bzw. B bestimmt wird
Mathespezialschüler Auf diesen Beitrag antworten »

Die dritte Aufgabe fand ich ziemlich einfach für ne Bundesrunde. Da hab ich ja 11. Klasse Landesrunde schon eine wesentlich schwerere gehabt.
@Sciencefreak
Erklär mal bitte genau die zweite Abschätzung! verwirrt
AD Auf diesen Beitrag antworten »

Zitat:
Original von Mathespezialschüler
Die dritte Aufgabe fand ich ziemlich einfach für ne Bundesrunde.

Das ist so ungewöhnlich nicht: Es gab sogar schon IMO-Aufgaben, die mit einem Einzeiler erledigt waren. Augenzwinkern
Sciencefreak Auf diesen Beitrag antworten »

Zitat:
Original von Mathespezialschüler
Die dritte Aufgabe fand ich ziemlich einfach für ne Bundesrunde. Da hab ich ja 11. Klasse Landesrunde schon eine wesentlich schwerere gehabt.
@Sciencefreak
Erklär mal bitte genau die zweite Abschätzung! verwirrt


Ich ka mit Geometrie noch nie so richtig klar außerdem sind meine Leistungen bei den Matheolympiaden meist besser, da ich mich dort voll auf die Aufgaben konzentriere

Zu meiner Abschätzung. Also erst mal kann ich nicht so gut erklären wie Arthur

Also meine Voraussetzungen sind erst mal nur, dass
wobei n wie in der Aufgabe eine natürliche Zahl ist, also positiv.
Dann gilt

Da dort Zahlen addiert werden die alle größer oder gleich (siehe obige Voraussetzung)
Nun gibt es für im Bereich von 2 bis n bestimmt viele dieser Intervalle und diese Zahl ist exakt der binäre Logarithmus von n. Also für n=2 gibt es ein einziges Intervall nämlich nur 2 für n=4 sind es dann schon 2 Intervalle nämlich zusätzlich noch das von 3 bis 4. Auch nichtnatürliche Lösungen vom binären Logarithmus sind richtig, da man in diesem Fall halt keinen ganzen Intervall betrachtet, sondern nur ein Teilstück und dementsprechend dann halt alle Nenner nur zu n werden lässt.
Aber das kann man auch weglassen, da es sich schwer begründen lässt denn der binäre Logarithmus von 1024^2 ist auch schon 20.
Mathespezialschüler Auf diesen Beitrag antworten »

Hättst das doch gleich gesagt! Augenzwinkern
Warum schreibst du dann ln, das war das, was mich verwirrt hat. Zumal sich der binäre Logarithmus doch viel besser berechnen lässt im Kopf! Egal. Also ich hab das jetzt so verstanden:

Sei , dann ist



Um es mal ordentlich vorm Gesicht zu haben Augenzwinkern
Aber ich sehe nicht, wie du das bekommen möchtest? verwirrt
AD Auf diesen Beitrag antworten »

Das geht z.B. so: Es ist für alle , nachweisbar durch Induktion (ähnlich wie bei e, mit Bernoulli-Ungleichung).

Daraus folgt , und jetzt noch Summation von 2 bis n .
Mathespezialschüler Auf diesen Beitrag antworten »

@Arthur
Nett! So hat das Sciencefreak aber sicher nicht gemacht (möcht ich mal behaupten Augenzwinkern ). Wahrscheinlich meinte Sciencefreak die n, die durch darstellbar sind. Denn für die größeren ist es dann ja auch klar, die Summe ist ja wachsend. Aber letztendlich ist es ja auch egal. Mit meiner Abschätzung reicht ja , also braucht man nur ein und dass die Aussage C erfüllt, ist ja schon geklärt.
Sciencefreak Auf diesen Beitrag antworten »

Zitat:
Original von Mathespezialschüler
Aber ich sehe nicht, wie du das

Wo liegt dein Problem? Du hast es doch oben ordentlich aufgeschreiben, du solltest nur beachten, dass für n=2 deine eine Abschätzung oben falsch ist, deshalb auch das =.
AD Auf diesen Beitrag antworten »

Der Einwand von MSS bezog sich sicher darauf, dass der Beweis zwar



liefert, nicht aber das stärkere

.

Für die vorliegende Aufgabe ist das natürlich völlig unerheblich, aber über die sind wir ja wohl in der Diskussion schon hinaus.
Sciencefreak Auf diesen Beitrag antworten »

Da einen ordentlichen Beweis für anstellen könnte etwas umständlich sein, aber es müsste doch eigentlich auch stimmen. Du kannst dir ja gerne einen Beweis überlegen, ich fahre morgen zur DeMO und werde dort genug Mathe machen. Ich berichte euch dann Mittwoch oder am Donnerstag(kommt drauf an, wann ich wiederkomme) wie es war
Neue Frage »
Antworten »



Verwandte Themen

Die Beliebtesten »
Die Größten »
Die Neuesten »